General Aptitude: School P

You might also like

Download as pdf or txt
Download as pdf or txt
You are on page 1of 57

Downloaded From : www.EasyEngineering.

net

|EC-2020|

GENERAL APTITUDE

1. The following figure shows the data of students enrolled in 5 years (2014 to 2018) for two schools P and
Q. During this period, the ratio of the average number of the students enrolled in school P to the average
of the difference of the number of students enrolled in schools P and Q is_________.
9
School P
8
Number of students (in thousands)

School Q
7

ww 6
5

w.E 4
3

2
asy
1
0 En
2014 2015 2016
Yeargin 2017 2018

(A) 23: 8 (B) 31: 23 eer


(C) 23:31 (D) 8:23
Key:
Exp:
(A)
From the figure,
i ng.
Number of students enrolled in school P from 2014 to 2018 are 3000, 5000, 5000, 6000, 4000
 Average number of students enrolled in school P
n et
3000 + 5000 + 5000 + 6000 + 4000 23000
= = = 4600 ... (1)
5 5
Difference of the number of students enrolled in schools P and Q from 2014 to 2018 are 1000, 2000,
3000, 1000, 1000
 Average of difference of the number of students enrolled in schools

1000 + 2000 + 3000 + 1000 + 1000 8000


P and Q = = = 1600 ... ( 2 )
5 5
 From (1) & ( 2 ) ,

The required ratio = 4600 :1600 = 46 :16 = 23: 8


© All rights reserved by Thinkcell Learning Solutions Pvt. Ltd. No part of this booklet may be reproduced or utilized in any form without the written permission.

Downloaded From : www.EasyEngineering.net


Downloaded From : www.EasyEngineering.net

|EC-2020|

2. It is quarter past three in your watch. The angle between the hour hand and the minute hand is _______.
(A) 0° (B) 15° (C) 22.5° (D) 7.5°
Key: (D)
Exp: The angle between the hands can be found using the formula:
1
Angle = ( 60H − 1) M ; where H is the hour, M is the minute
2
1 1
=
2
( 60 ( 3) − (11)(15) = (180 − 165) =
2
15
2
= 7.5

3. ww
The global financial crisis in 2008 is considered to be the most serious world-wide financial crisis,

w.E
which started with the sub-prime lending crisis in USA in 2007. The subprime lending crisis led to the
banking crisis in 2008 with the collapse of Lehman Brothers in 2008. The sub-prime lending refers to
the provision of loans to those borrowers who may have difficulties in repaying loans, and it arises

asy
because of excess liquidity following the East Asian crisis.
Which one of the following sequences shows the correct precedence as per the given passage?

En
(A) East Asian crisis → subprime lending crisis → banking crisis → global financial crisis.
(B) Subprime lending crisis → global financial crisis → banking crisis → East Asian crisis.

gin
(C) Global financial crisis → East Asian crisis → banking crisis → subprime lending crisis.

Key: (A) eer


(D) Banking crisis → subprime lending crisis → global financial crisis → East Asian crisis.

4. The Canadian constitution requires that equal importance be given to English and French.
i ng.
Last year, Air Canada lost a lawsuit, and had to pay a six-figure fine to a French-speaking couple after
they filed complaints about formal in-flight announcements in English lasting 15 seconds, as opposed to
informal 5 second messages in French.
n et
The French-speaking couple were upset at
(A) the English announcements being clearer than the French ones.
(B) the English announcements being longer than the French ones.
(C) equal importance being given to English and French.
(D) the in-flight announcements being made in English
Key: (A)

© All rights reserved by Thinkcell Learning Solutions Pvt. Ltd. No part of this booklet may be reproduced or utilized in any form without the written permission.

Downloaded From : www.EasyEngineering.net


Downloaded From : www.EasyEngineering.net

|EC-2020|

5. A superadditive function f ( g) satisfies the following property

f ( x1 + x 2 )  f ( x 1 ) + f ( x 2 )

Which of the following functions is a superadditive function for x>1?


(A) ex (B) e− x (C) x (D) 1 x
Key: (A)

6. Select the word that fits the analogy:


Explicit: Implicit:: Express:___________

Key:
ww
(A) Compress
(D)
(B) Suppress (C) Impress (D) Repress

7.
w.E
He was not only accused of theft_______of conspiracy.

asy
(A) but also (B) rather (C) but even (D) rather than
Key: (A)

8. En
a, b, c are real numbers. The quadratic equation ax 2 − bx + c = 0 has equal roots, which is , then

(A)  =
b
(B) b2  4ac gin
(C) 3 =
bc
( 2a 2 )
(D) 2 = ac

eer
a

Key: (D)
Exp: Given that; the quadratic equation ax 2 − bx + c = 0 has equal roots, which is 
 Let ,  be the roots of the equation
i ng.
Then  +  =

c
− ( −b )
a
=
b
a
→ (1) n et
 = ...( 2 )
a
b
From (1) ;  +  = Q  = 
a
b
 2 = ...( 3)
a

© All rights reserved by Thinkcell Learning Solutions Pvt. Ltd. No part of this booklet may be reproduced or utilized in any form without the written permission.

Downloaded From : www.EasyEngineering.net


Downloaded From : www.EasyEngineering.net

|EC-2020|

c
From ( 2 ) ;  . = c a  2 = ... ( 4 )
a
bc
( 3)  ( 4 )  23 =
a2
bc
 2 =
2a 2
9. A circle with centre O is shown in the figure. A rectangle PQRS of maximum possible area is inscribed
in the circle. If the radius of the circle is a, then the area of the shaded portion is_________.

P Q

ww O

w.E 

asy S R

(A) a 2 − 2a 2 (B) a 2 − 2a 2 En (C) a 2 − 3a 2 (D) a 2 − a 2


Key: (A)
Area of rectangle = ( 2x )( 2y )
gin
Exp:
 A = 4xy eer Y

 A 2 = 16x 2 y 2
 f ( x ) = 16x 2 ( a 2 − x 2 )
i ng.
 f ' ( x ) = 0  32xa 2 − 64x 3 = 0
 32x a 2 − 2x 2  = 0
y
x (0,0)
a n 2y
X
et
 2x = a 2 2
x
a −y
a
x=
2
2x
a S
x = only possible
2
a
If x = then
2

© All rights reserved by Thinkcell Learning Solutions Pvt. Ltd. No part of this booklet may be reproduced or utilized in any form without the written permission.

Downloaded From : www.EasyEngineering.net


Downloaded From : www.EasyEngineering.net

|EC-2020|

a2 a2
Q y 2 = a 2 − x 2  =
y2 = a 2 −
2 2
a
y=
2
a
y= only possible
2
a a
 Area of rectangle is maximum only when x = &y=
2 2
a a
 Area of rectangle =4xy = 4 = 2a 2
2 2

ww
Required shaded portion area = Area of circle - Area of rectangle
= a 2 − 2a 2 .

10. w.E
The untimely loss of life is a cause of serious global concern as thousands of people get killed_____

asy
accidents every year while many other die______ diseases like cardio vascular disease, cancer, etc.
(A) from, from (B) during, from (C) from, of (D) in, of
Key: (D)
En
gin
eer
i ng.
n et

© All rights reserved by Thinkcell Learning Solutions Pvt. Ltd. No part of this booklet may be reproduced or utilized in any form without the written permission.

Downloaded From : www.EasyEngineering.net


Downloaded From : www.EasyEngineering.net

|EC-2020|

ELECTRONICS AND COMMUNICATION ENGINEERING

Q. No. 1 to 25 Carry One Mark Each

1. The components in the circuit shown below are ideal. If the op-amp is in positive feedback and the input
voltage Vi is a sine wave of amplitude 1V, the output voltage V0 is
1k

ww 1V 1k
+5V

−1V
0
w.E Vi
+

V0

asy −5V

(A) a non-inverted sine wave of 2V amplitude. En


(B) a constant of either +5V or -5V.
gin
(C) an inverted sine wave of 1 V amplitude.
(D) a square wave of 5 V amplitude eer
Key:
Sol:
(B)
Given, i ng.
V1 = 1sin ( t )( Volts )
V0 = ? R 2 = 1k n et
R1 = 1k + Vsat = 5
Vi +
V+ Vo

−Vsat = −5

For the above circuit to act as Schmitt trigger VTL (Lower threshold voltage) and VTH (Higher threshold
voltage). Values should not go beyond positive peak and negative peak values of “V 1”.

© All rights reserved by Thinkcell Learning Solutions Pvt. Ltd. No part of this booklet may be reproduced or utilized in any form without the written permission.

Downloaded From : www.EasyEngineering.net


Downloaded From : www.EasyEngineering.net

|EC-2020|

Threshold voltages for the above circuit is given by


R1
VTL = −Vsat  = −5V
R2

R1
And VTH = Vsat  = +5V
R2
We got VTL and VTH goes beyond 1V (positive and negative peak values of V1) the above circuit
can’t acts as a Schmitt trigger, so, it cannot convert the input sine wave to square wave.
 In the above circuit there is no scope for the output to change from + Vsat ( 5V ) to − Vsat ( −5V ) (or) vice
versa.

ww
Hence the output will remain either at +5V (or) –5V.
Hence the correct option is (B).

2. w.E
The partial derivative of the function

f ( x, y,z ) = e1− x cos y + xze


asy
(
−1 1+ y2

With respect to x at the point (1, 0, e) is


)

(A)
1
(B) 1 En (C) –1 (D) 0

Key: (D)
e
gin
Exp: f ( x, y,z ) = e1− x cos y + xze
−1
(1+ y ) 2

eer
f
x
= e1− x cos y  − cos y  + z e1+ y
2
−1
i ng.

f
x (1,0,e )
−1
= e1−1.cos( 0 )  − cos (0)  + e.e1+ 0

= e0 [−1] + e.e −1 = −1 + 1 = 0
n et
f
 =0
x (1,0,e )

d2 y dy
3. The general solution of 2
− 6 + 9y = 0 is
dx dx
(A) y = C1e3x + C2e−3x (B) y = ( C1 + C2 x ) e3x

(C) y = ( C1 + C2 x ) e−3x (D) y = C1e3x

Key: (B)

© All rights reserved by Thinkcell Learning Solutions Pvt. Ltd. No part of this booklet may be reproduced or utilized in any form without the written permission.

Downloaded From : www.EasyEngineering.net


Downloaded From : www.EasyEngineering.net

|EC-2020|

Sol: Given D.E is


d2 y dy
2
− 6 + 9y = 0
dx dx
 ( D − 6D + 9 ) y = 0
2

 Auxiliary equation is m2 − 6m + 9 = 0
 ( m − 3) = 0  m = 3,3 real & repeated roots
2

The required solution is


y = e3x ( C1 + C2 x )

4. ww
The loop transfer function of a negative feedback system is
K ( s + 11)
G (s) H (s ) =
w.E s ( s + 2 )( s + 8 )

Key: (160) asy


The value of K, for which the system is marginally stable, is ________.

Sol: Given,
k ( s + 11) En
G (s) H (s ) =
s ( s + 2 )( s + 8 )
gin
and we need to find the value of ‘k’ for marginally stable system.

1 + G (s ) H (s ) = 0

 s ( s + 2 )( s + 8 ) + k ( s + 11) = 0
eer
 s ( s 2 + 10s + 16 ) + ks + 11k = 0
 s3 + 10s 2 + (16 + k ) s + 11k = 0
i ng.
Since, it is 3rd order characteristics equation,
Internal product = External product (for marginally stable system)
n et
 10 (16 + k ) = 11k
 160 + 10k = 11k
 160 = k
k max = 160
Hence, the value of ‘k’ for marginally stable is 160.

© All rights reserved by Thinkcell Learning Solutions Pvt. Ltd. No part of this booklet may be reproduced or utilized in any form without the written permission.

Downloaded From : www.EasyEngineering.net


Downloaded From : www.EasyEngineering.net

|EC-2020|

5. For a vector field A , which one of the following is FALSE?

(A)   A is another vector field.

(B) A is irrotational if  2 A = 0.

( ) ( )
(C)     A =  .A −  2 A

(D) A is solenoidal if .A = 0.


Key: (B)

6. In the circuit shown below, all the components are ideal, and the input voltage is sinusoidal. The

ww
magnitude of the steady-state output V0 (rounded off to two decimal places) is ______V.

w.E +
C1 = 0.1F D2

230 V ( rms ) asy D1 C2 = 0.1F V

En
0

Key: (650.538) gin


Sol: Given circuit
eer
+
C1

+
i + ng.
230V
rms
D1 Vo
D2

C2 V0 n et
− − −

Positive clamper Positive peak detector

The above circuit acts as voltage doubler circuit


 Vo = 2  Peak value of the input voltage ( VP )

Peak value of the input voltage VP = 230 2

 VO = 2  230 2 = 650.538 ( volts )

© All rights reserved by Thinkcell Learning Solutions Pvt. Ltd. No part of this booklet may be reproduced or utilized in any form without the written permission.

Downloaded From : www.EasyEngineering.net


Downloaded From : www.EasyEngineering.net

|EC-2020|

7. In the circuit shown below, all the components are ideal. If Vi is + 2V, the current Io sourced by the op-
amp is _________mA.
1k

5V
1k
− Io

Vi +
1k

ww −5V

Key: (6) w.E


Sol: Given,

asy R f = 1k I2

R1 = 1k 5V
En
I1 N 0A

Io O
gin Vo
Vi
P
0A
+

−5V
IL
eer
R L = 1k

i ng.
From virtual short concept VN = VP = Vi
Applying KCL at node “N” we get
I1 = I2
n et
0 − VN VN − V0
= where VN = Vi
R1 Rf

 R 
 Vo = 1 + f  Vi
 R1 
 Vo = 2Vi
Applying KCL at node “O” we get

© All rights reserved by Thinkcell Learning Solutions Pvt. Ltd. No part of this booklet may be reproduced or utilized in any form without the written permission.

Downloaded From : www.EasyEngineering.net


Downloaded From : www.EasyEngineering.net

|EC-2020|

I2 + I0 = I L
VN − V0 V
+ I0 = 0
Rf RL
Vi − 2Vi 2V
+ I0 = i  V0 = 2Vi and VN = Vi 
1k 1k
3V 3  2
 I0 = i = 3 = 6mA
1k 10

8. A binary random variable X takes value +2 or -2. The probability P ( X = +2 ) = . The value of 
(rounded off to one decimal place), for which the entropy of X is maximum, is ________.

Key: ww
(0.5)
Sol:
w.E
Given P ( X = +2 ) = 
P ( X = −2 ) = 1 − 

asy
H ( X ) = − log 2  − (1 −  ) log 2 (1 −  )
dH ( X )
d
= − log 2  + log 2 (1 −  )
En
The maximum value of H(X) requires that
dH ( X )
=0
gin
d
(1 −  ) = 0 H (X)
eer
log 2

1− 


= 20 = 1
i ng.
1−  = 
2 = 1
1
n et
=
2 0 1 1 
  = 0  H (X) = 0 2
 = 1 H (X) = 0
1 1 1
=  H ( X ) = log 2 2 + log 2 2 = 1 bit symbol
2 2 2

© All rights reserved by Thinkcell Learning Solutions Pvt. Ltd. No part of this booklet may be reproduced or utilized in any form without the written permission.

Downloaded From : www.EasyEngineering.net


Downloaded From : www.EasyEngineering.net

|EC-2020|

9. The figure below shows a multiplexer where S1 and S0 are the select lines, I0 to I3 are the input data
lines, EN is the enable line, and F (P, Q, R) is the output. F is

0 EN
R I0
0 I1 MUX F
R I2
1 I3 S1 S0

P Q

ww
(A) PQR + PQ. (B) P + QR (C) Q + PR (D) PQ + QR
Key:
Sol:
(D)
w.E
In this case we need to obtain the output expression of the multiplexer circuit given below

asy
R
0
EN
En
R
gin
F

1 S1 S0
eer
F = PQR + PQ.0 + PQR + PQ.1
P Q
i ng.
= PQR + PQR + PQ
(
= QR P + P + PQ ) n et
= PQ + QR
Hence the correct answer is option (D).

10. Which one of the following pole-zero plots corresponds to the transfer function of an LTI system
characterized by the input-output difference equation given below?
3
y  n  =  ( −1) x  n − k 
k

k =0

© All rights reserved by Thinkcell Learning Solutions Pvt. Ltd. No part of this booklet may be reproduced or utilized in any form without the written permission.

Downloaded From : www.EasyEngineering.net


Downloaded From : www.EasyEngineering.net

|EC-2020|

(A) 1m (B) 1m

1 1
rd
3 order pole 4th order pole

−1 1
−1 
1  Re
Re

−1 −1

ww 1m
1m
(C)

w.E 1
3rd order pole
(D)
1
3rd order pole

asy −1

En
1
−1 
1  Re
Re

gin
−1
eer −1

Key:
Sol:
(D)
The given equation is i ng.
3
y ( n ) =  ( −1) x ( n − k )
k =0
k

We need to obtain the pole zero plot of its corresponding transfer function
n et
y ( n ) = ( −1) x ( n − 0 ) + ( −1) x ( n − 1) + ( −1) x ( n − 2 ) + ( −1) x ( n − 3 )
0 1 2 3

 Y ( z ) = X ( z ) − z −1X ( z ) + z −2 X ( z ) − z −3X ( z ) ( x (n − n )  z 0
− n0
X (z ) )
Y (z)
 = 1 − z −1 + z −2 − z −3
X (z)

1 1 1  Y (z) 
 H (z) = 1 − + 2 − 3  = H ( z ) 
z z z  X (z) 
z3 − z 2 + z − 1 ( z − 1) ( z + 1)
2

= =
z3 z3

© All rights reserved by Thinkcell Learning Solutions Pvt. Ltd. No part of this booklet may be reproduced or utilized in any form without the written permission.

Downloaded From : www.EasyEngineering.net


Downloaded From : www.EasyEngineering.net

|EC-2020|

So, the transfer function having 3 zeros located at z = 1, z = + j, z = –j and it has 3 poles and all of them
lies on origin. So, the pole-zero plot is as shown below
Img

3rd order pole

× Re

Unit circle

ww
w.E
Hence the correct answer is option (B).

11. A transmission line of length 3 4 and having a characteristic impedance of 50 is terminated with a

asy
load of 400. The impedance (rounded off to two decimal places) seen at the input end of the
transmission line is________  .
Key: (6.25)
En
Sol: Given, Zo = 50

3 gin
Length of transmission line

Load impedance ZL = 400


=
4
m.
eer
 3 
Input impedance Zin  =  = ?
i Zo = 50
ng. ZL = 400


Zin  =
 4


 ZL
4 

( 2m + 1)   = Zo2
=
3
4
n =0
et
Where m = 0, 1, 2 ….. Zin = ?
 3  Z2 50  50
 Zin  =  = o = = 6.25
 4  ZL 400

 3 
input impedance Zin  =  = 6.25
 4 

© All rights reserved by Thinkcell Learning Solutions Pvt. Ltd. No part of this booklet may be reproduced or utilized in any form without the written permission.

Downloaded From : www.EasyEngineering.net


Downloaded From : www.EasyEngineering.net

|EC-2020|

12. A single crystal intrinsic semiconductor is at a temperature of 300K with effective density of states for
holes twice that of electrons. The thermal voltage is 26 mV.
The intrinsic Fermi level is shifted from mid-bandgap energy level by
(A) 9.01 meV (B) 18.02 meV (C) 13.45 meV (D) 26.90 meV

Key: (A)
1 N 
Sol: E Fi − E midgap = kTl n  V 
2  NC 
Given, NV = 2NC

ww
In question they asked, EFi = Emid + 9.01 meV

13. w.E
In an 8085 microprocessor, the number of address lines required to access a 16 K byte memory bank

Key:
Is _______.
(14) asy
Sol:
En
In this case we need to obtain number of address lines needed to access a 16 kB memory using 8085 p.

16kB = 16k 1B = 24  210  8 = 214  8 = 2P  m


So, P = 14 i.e., we need minimum 14 address line to access the memory chip. gin
eer
i
14. The random variable
 1; 5  t  7
Y =  W ( t )  ( t ) dt, where  ( t ) = 
−
0; otherwise
ng.
And W (t) is a real white Gaussian noise process with two-sided power spectral density
Sw ( f ) = 3W Hz, for all f. The variance of Y is __________.
n et
Key: (6)

Sol: Given Y =  W ( t )  ( t )dt
−

(i)  y = E  y

(ii) 2 = E ( Y −  y ) 
2

 
(iii)  ( t ) = 1, 5 t 7

© All rights reserved by Thinkcell Learning Solutions Pvt. Ltd. No part of this booklet may be reproduced or utilized in any form without the written permission.

Downloaded From : www.EasyEngineering.net


Downloaded From : www.EasyEngineering.net

|EC-2020|

  
E  Y = E   ( W ( t )  ( t ) ) dt  =  E  W ( t )  ( t ) dt = 0
 −  −
 R W (  ) = 3 (  )

E ( Y − 0 )  = E  Y 2 
2
 

 

= E   ( W ( t ) )  ( t )dt.  W ( t ' )  ' ( t ) dt 
 − − 
 
=   E  W ( t ).W ( t ') . ( t ')  ( t ) dt
− −

ww
 
=   R ( t '− t )  ' ( t )  ( t ) dt
− −
W

w.E
 
=   3 ( t '− t )  ' ( t ). ( t )dt
− −

= 3  2 ( t ) dt
−
7 asy
= 31 dt
5
En
= 3 2 = 6
gin
15.
eer
A digital communication system transmits a block of N bits. The probability of error in decoding a bit is
. The error event of each bit is independent of the error events of the other bits. The received block is

i
declared erroneous if at least one of its bits is decoded wrongly. The probability that the received block
is erroneous is
ng.
Key:
Sol:
(A) N (1 −  )

(C)
N-bits transmitted independently,
(B) 1−  N
(C) 1 − (1 − )
N
(D)  N
n et
Y = Probability of no-error = (1 −  )(1 −  ) ... N times
= (1 −  )
N

Probability of received block is erroneous is

1 − Y = 1 − (1 −  ) .
N

© All rights reserved by Thinkcell Learning Solutions Pvt. Ltd. No part of this booklet may be reproduced or utilized in any form without the written permission.

Downloaded From : www.EasyEngineering.net


Downloaded From : www.EasyEngineering.net

|EC-2020|

16. A 10-bit D/A converter is calibrated over the full range from 0 to 10 V. If the input to the D/A converter
is 13A (in hex), the output (rounded off to three decimal places) is ________V.
Key: (3.066)
Sol: It is given that for a 10-bit DAC, the voltage range is (0-10) V, we need to obtain the output for input
(13A)16.
VFSO 10
Resolution = = 10
2n 2
Analog output = Resolution × (Decimal equivalent of input)
10
=  314  (13A )16 = ( 314 )10 
210

ww = 3.066
Hence the correct answer is 3.066V.

17.
w.E  40 60 
In the given circuit, the two-port network has the impedance matrix  Z =   . The value of Z L

asy
for which maximum power is transferred to the load is _________  .
 60 120

10 I1
En I2

120V +
− V1
+
 Z gin +
V2 ZL

eer −

Key: (48) i ng.


Sol: The given circuit is as shown below

10 I1 I2
n et
+ +
 40 60 
120V + V1 Z= 
−  60 120 
V2 ZL
− −

We need to obtain Z L so that it will receive maximum power from the circuit
Using Thevenin theorem the above circuit can be replaced by Thevenin equivalent as show below

© All rights reserved by Thinkcell Learning Solutions Pvt. Ltd. No part of this booklet may be reproduced or utilized in any form without the written permission.

Downloaded From : www.EasyEngineering.net


Downloaded From : www.EasyEngineering.net

|EC-2020|

Zth I2

+
+ ZL
Vth − V2

By maximum power transfer theorem, for maximum power transfer to ZL , the condition is ZL = Zth .
To obtain Zth of the 2 port network given from output port, the independent voltage source (120V)

ww
should be short circuited, and then ratio of V2 to I 2 is Zth .

w.E 10

+
 40 60 
Z=
I2

+
I1 V1

asy 
 60 120  − V2

En
gin
The Z matrix equation is
V1 = 40I1 + 60I2 …(1)
V2 = 60I1 + 120I2 …(2)
From the circuit V1 = −10I1 , the equation 1 becomes eer
−10I1 = 40I1 + 60I 2
 −50I1 = 60I 2
i ng.
 I1 =
−6
5
I2 n et
Now equation 2 becomes
 −6 
V2 = 60  I 2  + 120I 2
 5 
V
 2 = −72 + 120
I2
 Zth = 48 = ZL
So, ZL = 48
Hence the correct answer is 48.

© All rights reserved by Thinkcell Learning Solutions Pvt. Ltd. No part of this booklet may be reproduced or utilized in any form without the written permission.

Downloaded From : www.EasyEngineering.net


Downloaded From : www.EasyEngineering.net

|EC-2020|

18. The output y[n] of a discrete-time system for an input x[n] is


y  n  = max x  k .
− k  n

The unit impulse response of the system is


(A) unit step signal u [n]. (B) unit impulse signal   n .

(C) 1 for all n. (D) 0 for all n.


Key: (A)
Sol: It is given that
y ( n ) = max x ( k )
− k  n

ww
We need to obtain its unit impulse response.
If input is unit impulse i.e. x ( k ) =  ( k ) then output is

w.E
Impulse response y ( n ) = h ( n )

So replacing x ( k ) by  ( k ) and y(n) by h(n) in above equation we have

h ( n ) = max  ( k ) asy
En
− k  n

h ( −2 ) = max  ( k )
− k −2

= max   ( − ) ,....  ( −3) ,  ( −2 ) 


= max  0 , 0 , 0 .. 0  = 0 gin
From this we can say if n  −1 then h ( n ) = 0 eer
If n  0 then h ( n ) = 1 for all n,

h ( 5 ) = max  ( k )
i ng.
− k  5

= max ...  ( −11) ,  ( 0 ) ,  (1) ,  ( 2 ) ,  ( 3) ,  ( 4 ) ,  ( 5 ) 


= max ... 0 , 1 , 0 , 0 , 0 , 0 , 0  = max 0,1,0,0,0 = 2
n et
Overall h ( n ) = 0; n  0
h ( n ) = 1; n  0
 h ( n ) = u ( n ) i.e., unit step signal

Hence the correct answer is option (A).

© All rights reserved by Thinkcell Learning Solutions Pvt. Ltd. No part of this booklet may be reproduced or utilized in any form without the written permission.

Downloaded From : www.EasyEngineering.net


Downloaded From : www.EasyEngineering.net

|EC-2020|

19. If v1 , v2 ,...., v6 are six vectors in R 4 , which one of the following statements is FALSE?

(A) If v1 , v3 , v5 , v 6  spans R 4 , then it forms a basis for R 4 .

(B) Any four of these vectors form a basis for R 4 .


(C) These vectors are not linearly independent.
(D) It is not necessary that these vectors span R 4 .
Key: (B)
Exp: Given
V1 , V2 , ....,V6 are six vectors in R 4

ww
For a 4-dimensional vector space,
(i) any four linearly independent vectors form a basis (or)

w.E
(ii) Any set of four vectors in R 4 spans R 4 , then it forms a basis.
 Clearly options (A), (C), (D) are true.
 option (B) is FALSE.
asy
20. In the circuit shown below, the Thevenin voltage VTH is
En
2V
−+
2
gin 4
+

1A 1 2A eer 2 VTH

i − ng.
(A) 4.5V (B) 2.4V (C) 3.6V (D) 2.8V n et
Key: (C)
Sol: The given circuit is as shown below
2V
2 4
−+ +

1A 1 2A 2 Vth

© All rights reserved by Thinkcell Learning Solutions Pvt. Ltd. No part of this booklet may be reproduced or utilized in any form without the written permission.

Downloaded From : www.EasyEngineering.net


Downloaded From : www.EasyEngineering.net

|EC-2020|

We need to obtain the value of Vth.


If we do source transformation to above practical current sources, then the circuit becomes

2V
2 4
−+ +
1 2
Vth
+ Ix
− 1V +
− 4V

ww
Ix =
1 + 2 − 4 −1
1+ 2 + 2 5
= A

−2
Vth = 2I x + 4 =

So Vth = 3.6V w.E


5
+ 4 = 3.6V

Hence the correct answer is option (C).


asy
21.
En
The current in the RL-circuit shown below is i ( t ) = 10 cos ( 5t −  4 ) A.

The value of the inductor (rounded off to two decimal places) is ________H.
gin
R
eer
i(t)

200cos ( 5t ) V ~
+
i L ng.

n et
Key: (2.82)
Sol: The given circuit is shown below
R
 
i ( t ) = 10cos  5t − 
 4
200cos5t ~ L

© All rights reserved by Thinkcell Learning Solutions Pvt. Ltd. No part of this booklet may be reproduced or utilized in any form without the written permission.

Downloaded From : www.EasyEngineering.net


Downloaded From : www.EasyEngineering.net

|EC-2020|

We need to obtain the value of inductor L


The source voltage and the loop current is known, so the impedance of the network is
V 2000o 20 20
Z= = = 2045 = +j = R + jX L
I 10 − 45o 2 2
20 20 20
by comparision X L =  L = L= (  = 5)
2 2 5 2
 L = 2.82H
So, the value of inductor is 2.82H

22.
ww
The impedances Z = jX, for all X in the range ( −,  ) , map to the Smith chart as

w.E
(A) a line passing through the centre of the chart.
(B) a point at the centre of the chart.
(C) a circle of radius 0.5 with centre at (0.5,0).

asy
(D) a circle of radius 1 with centre at (0, 0).
Key:
Sol:
(D)
Given,
En
Load impedance Z = jX (purely reactive)
gin
On the smith chart, as you move on the constant resistance “R” = 0 circle whose centre is (0, 0) and
radius is “1”, the impedance seen on this circle is purely reactive.
eer
Hence the correct option is (D).

i ng.
23. Consider the recombination process via bulk traps in a forward biased pn homojunciton diode. The
n et
maximum recombination rate is U max . If the electron and the hole capture cross-sections are equal,
which one of the following if FALSE?
(A) With all other parameters unchanged, Umax decreases if the intrinsic carrier density is reduced.

(B) Umax depends exponentially on the applied bias.

(C) With all other parameters unchanged, Umax increases if the thermal velocity of the carriers
increases.
(D) Umax occurs at the edges of the depletion region in the device
Key: (D)

© All rights reserved by Thinkcell Learning Solutions Pvt. Ltd. No part of this booklet may be reproduced or utilized in any form without the written permission.

Downloaded From : www.EasyEngineering.net


Downloaded From : www.EasyEngineering.net

|EC-2020|

qV
1
Sol: u max = 0 VTh N t n i e 2kT
2
(i) u max VTh
(ii) u max n i
qV
(iii) u max e 2kT
 And also u max occurs in the depletion region when (n + P) is minimum, not at the edges of depletion.

24. The two sides of a fair coin are labelled as 0 and 1. The coin is tossed two times independently. Let M

Key:
ww
and N denote the labels corresponding to the outcomes of those tosses. For a random variable X, defined
as X=min (M,N) , the expected value E(X) (rounded off to two decimal places) is ______.
(0.25)
Sol:
w.E
Given that, the two sides of a fair coin are labelled as 0 and 1. Let M and N denote the labels
corresponding to the outcomes of those tosses.
Random variable
X = min ( M, N ) asy
 X = min ( 0,0 ) = 0; min ( 0,1) = 0;min (1,0 ) = 0;min (1,1) = 1
En
X = 0,1. ( Discrete r.v )

 Probability distribution table is gin


X 0 1
eer
P(X)
3
4
1
4
i ng.
3
4
 Expected value E ( X ) = 0.25
1 1
 E ( X ) =  x P ( x ) = 0. + 1. = = 0.25
4 4
n et

© All rights reserved by Thinkcell Learning Solutions Pvt. Ltd. No part of this booklet may be reproduced or utilized in any form without the written permission.

Downloaded From : www.EasyEngineering.net


Downloaded From : www.EasyEngineering.net

|EC-2020|

25. The pole-zero map of a rational function G(s) is shown below. When the closed contour  is mapped
into the G(s)- plane, then the mapping encircles

1m

S − plane

o x

−1
   Re

ww o

w.E −1
x

asy
En
(A) the point -1+j0 of the G(s)-plane once in the clockwise direction.
(B) the point -1+j0 of the G(s)-plane once in the counter-clockwise direction.
(C) the origin of the G(s)-plane once in the clockwise direction
(D) the origin of the G(s)-plane once in the counter-clockwise direction. gin
Key: (D)
eer
Sol: Given, P = 4, Z = 3
We need to find the mapping of G(s), N = P – Z = 4 – 3 = 1 i ng.
Hence, origin of G(s) are in CCW.

n et

© All rights reserved by Thinkcell Learning Solutions Pvt. Ltd. No part of this booklet may be reproduced or utilized in any form without the written permission.

Downloaded From : www.EasyEngineering.net


Downloaded From : www.EasyEngineering.net

|EC-2020|

Q. No. 26 to 55 Carry Two Marks Each


X ( ) is the Fourier transform of x(t) shown below. The value of  X ( ) d (rounded off to two
2
26.
−

decimal places) is______________.


x(t)
3

ww
1

−3 t
−2 −1

w.E 0 1 2 3 4

Key: (58.61)
Sol: The signal x(t) is as shown below
asy x(t)
3
En
gin
1 eer
−1 0 1 2 i 3
t
ng.
We need to obtain value of

 x ( )
2
d
n et
−

By Parseval’s theorem
  

 X ( ) d = 2  x ( t ) dt = 2  ( x ( t ) ) dt
2 2 2

− − −

[Since the given time domain signal x(t) is real, then the mod function does not have any role].
We know that shifting does not change the energy of signal so if we define y ( t ) = x ( t + 1) , it means
energy of x(t) and y(t) are same. Now y(t) becomes

© All rights reserved by Thinkcell Learning Solutions Pvt. Ltd. No part of this booklet may be reproduced or utilized in any form without the written permission.

Downloaded From : www.EasyEngineering.net


Downloaded From : www.EasyEngineering.net

|EC-2020|

y ( t ) = x ( t + 1)
3

−1

t
−2 −1 0 1 2

ww
  

 X ( ) d = 2  x 2 ( t ) dt = 2  y 2 ( t ) dt
2

− − −

w.E
2 2 0
= 2  y 2 ( t ) dt = 2  2 y 2 ( t )dt = 4  y 2 ( t ) dt
−2 0 −2

( y ( t ) is even then y 2 ( t ) is also even, for even signal ) 



a a 0
asy
  y e ( t ) dt = 2  y e ( t ) dt = 2  y e ( t ) dt



−a 0 −a 
En

 X ( )
−
2 0 
d = 4   y 2 ( t ) dt 
 −2  gin
 −1 0
= 4   ( t + 2 ) dt +  ( 2t + 3) dt 
 −2
2 2 
 eer
i
−1

 −1

 −2
0

= 4   ( t 2 − 4t + 4 ) dt +  ( 4t 2 + 12t + 9 ) dt 
 ng.
n
−1

  t 3 −
 
et
1 0
t2   4t 3 t2
= 4   + 4 + 4t  +  + 12 + 9t  
  3 2  −2  3 2  −1 

1 4 
= 4  ( −1 + 8 ) + 2 (1 − 4 ) + 4 ( −1 + 2 ) + ( 0 + 1) + 6 ( 0 − 1) + 9 ( 0 + 1) 
3 3 
2 4  11 
= 4  − 6 + 4 + − 6 + 9  = 4  + 1
3 3  3 
 14  56
= 4   = = 58.61
 3 3
Hence the correct answer 58.6%.

© All rights reserved by Thinkcell Learning Solutions Pvt. Ltd. No part of this booklet may be reproduced or utilized in any form without the written permission.

Downloaded From : www.EasyEngineering.net


Downloaded From : www.EasyEngineering.net

|EC-2020|

27. A one-sided abrupt pn junction diode has a depletion capacitance C D of 50 pF at a reverse bias of 0.2V.
The plot of 1 C2D versus the applied voltage V for this diode is a straight line as shown in the figure
below. The slope of the plot is _____________ 1020 F−2 V−1.

1 C2D

ww 0
V

(A) –3.8 w.E (B) –1.2 (C) –5.7 (D) –0.4


Key:
Sol:
(A)
Given cD = 50PF, VR = 0.2V asy
 1 
  =+
2
2
( Vbi + VR ) En
 CD 

slope = −
q s N

2  1 
= −
2
1 gin
q s N

 CD  ( Vbi + VR )
eer
If Vbi = 0.85V then, we will get
slope = −3.8  1020 F−2 V −1 ,Option A is correct i ng.
28. In the voltage regulator shown below, V1 is the unregulated input at 15V. Assume VBE = 0.7V and the
base current is negligible for both the BJTs. If the regulated output Vo is 9V, the value of R 2 is
n et
_________  .
V1 = 15V Vo = 9V

R 3 = 1k

R1 = 1k

R2
VZ = 3.3V

© All rights reserved by Thinkcell Learning Solutions Pvt. Ltd. No part of this booklet may be reproduced or utilized in any form without the written permission.

Downloaded From : www.EasyEngineering.net


Downloaded From : www.EasyEngineering.net

|EC-2020|

Key: (800)
Sol: Given the following circuit
VBEON = 0.7V;  is very large implies IB  0A

R2 = ?

15V Vo = 9V

R3

R1 = 1k

ww + VL
I

w.E − 0.7
+
VZ = 3.3V
IB  0A IL
R2

asy − loop (1)

En
Apply KVL around loop (1) we get
VL − 0.7 − 3.3 = 0 gin
 VL = 4V
Applying KCL at node “L” we get eer
I = IL
V − VL 9 − 4
i ng.
Where I = 0
R1
 IL = I = 5mA
=
1k
= 5mA
n et
VL 4
R2 = = = 800
IL 5  10−3

29. Consider the following closed loop control system

R (s) + G (s)
C (s) Y(s)

© All rights reserved by Thinkcell Learning Solutions Pvt. Ltd. No part of this booklet may be reproduced or utilized in any form without the written permission.

Downloaded From : www.EasyEngineering.net


Downloaded From : www.EasyEngineering.net

|EC-2020|

1 s +1
where G ( s ) = and C ( s ) = K If the steady state error for a unit ramp input is 0.1, then the
s ( s + 1) s+3
value of K is _________.
Key: (30)
Sol: Given,

R (s) Output
+ E (s) G (s)

ww k ( s + 1)
G (s) =
1
w.E
s ( s + 1)
and E ( s ) =
( s + 3)
and ess = 0.1 for ramp

We need to find k value

ess =
1
and asy
k v = limsG ( s ) E ( s )
kv

k v = lim
s →0

s 1
k
( s + 1) En
k
s →0 s ( s + 1) ( s + 3)
gin
kv =

0.1 =
3
3 eer
k=
k
3
= 30 i ng.
Hence, the value of k is 30.
0.1

n et

© All rights reserved by Thinkcell Learning Solutions Pvt. Ltd. No part of this booklet may be reproduced or utilized in any form without the written permission.

Downloaded From : www.EasyEngineering.net


Downloaded From : www.EasyEngineering.net

|EC-2020|

30. The state diagram of a sequence detector is shown below. State S0 is the initial state of the sequence
detector. If the output is 1, then
00
10 00

S0 S1 S2 S3
00 10 00

ww
10 10
01
S4

w.E 10

(A) the sequence 01010 is detected


asy (B) the sequence 01110 is detected.

Key:
(C) the sequence 01001 is detected
(A) En (D) the sequence 01011 is detected

Sol:
gin
The state diagram of a sequence detector is given where the initial state is S0. If output is 1 then we need
to obtain the input sequence.
In general
eer
input

10
output
i ng.
n
SA SB

This transition means the state will transit from SA to SB when the input is 1 and at the end of transition it
will produce output as 0.
et
The given state diagram is
10 00

00 10 00
S0 S1 S1 S3
Initial state

10 10

S4
10 Final target

© All rights reserved by Thinkcell Learning Solutions Pvt. Ltd. No part of this booklet may be reproduced or utilized in any form without the written permission.

Downloaded From : www.EasyEngineering.net


Downloaded From : www.EasyEngineering.net

|EC-2020|

The above dotted line shows the desired sequence to get output 1. So, the state transition sequence is
S0 ⎯⎯
00
→S1 ⎯⎯
10
→S2 ⎯⎯
00
→S3 ⎯⎯
10
→S4 ⎯⎯
01
→S3
The corresponding input sequence is 0, 1, 0, 1, 0
The corresponding output sequence is 0, 0, 0, 0, 1
So, the input sequence detected is 01010
Hence the correct answer is option (A)

31. The characteristic equation of a system is

ww
s3 + 3s 2 + ( K + 2 ) s + 3K = 0

In the root locus plot for the given system, as K varies from 0 to  , the break-away or break-in point(s)
lie within
(A) ( −, − 3)w.E (B) ( −3, − 2 ) (C) ( −1, 0 ) (D) ( −2, − 1)
Key: (C)
asy
En
Sol: Given,
s + 3s 2 + ( k + 2 ) s + 3k = 0
We need to calculate break away / break in point
 s3 + 3s 2 + ks + 2s + 3k = 0 gin
 s + 3s + 2s + ( s + 3) k = 0
3 2

eer
1+
( s + 3) k
s + 3s 2 + 2s
2

− ( s3 + 3s 2 + 2s )
i ng.
k=
( s + 3)

For breakaway|breakin we need to find


dk
n et
ds

dk  ( 3s 2 + 6s + 2 ) ( s + 3) − s3 + 3s 2 + 2s 
 = − 
( s + 3)
2
ds  
 ( 3s3 + 6s 2 + 2s + 9s 2 + 18s + 6 − s3 − 3s 2 − 2s ) 
= 
( s + 3)
2
 
− 2s3 + 2s 2 + 18s + 6
=
( s + 3)
2

© All rights reserved by Thinkcell Learning Solutions Pvt. Ltd. No part of this booklet may be reproduced or utilized in any form without the written permission.

Downloaded From : www.EasyEngineering.net


Downloaded From : www.EasyEngineering.net

|EC-2020|

dk
= 0  s = −0.46, − 3.87, − 1.65
ds
Since –0.46 is a valid point. Hence correct option is (C).

32. For the given circuit, which one of the following is the correct state equation?

0.5H

i
+
i1 2 v 0.25F 1 i2

ww
(A)
w.E
d  v   −4 −4   v   4 4  i1 
= +  
dt  i   −2 4   i   0 4  i 2 
(B)
d  v   −4 −4   v   4 0  i1 
= +  
dt  i   −2 −4   i   0 4  i 2 

(C)
d  v   −4 4   v   0 4  i1 
= +
asy  
dt  i   −2 −4   i   4 0  i 2 
(D)
d  v   4 −4   v   0 4  i1 
= +  
dt  i   −2 −4   i   4 4  i 2 

Key: (C)
En
Sol: Given,
A i V gin
0.5H
P
+
v eer
i1 2
0.25F −
i
1 i2

ng.
We need to select the correct state equation of given circuit diagram
KCL at P point
n et
dv
i = 0.25 + V − i2
dt
dv
= −4V + 4i 2 + 4i = −4V + 4i + 4i 2 ...(i)
dt
KCL at point A

© All rights reserved by Thinkcell Learning Solutions Pvt. Ltd. No part of this booklet may be reproduced or utilized in any form without the written permission.

Downloaded From : www.EasyEngineering.net


Downloaded From : www.EasyEngineering.net

|EC-2020|

VA  di 
i1 = +i VA = V + 0.5 
2  dt 
di
V + 0.5
i1 = dt + i
2
di
 2i1 = V + 0.5 + 2i
dt
di
0.5 = −2i − V + 2i1
dt
di
= −4i − 2V + 4i1 ...(ii)
dt

ww
State equation
d  v   −4 4   V   0 4   i1 
= +
dt i   −2 −4   i   4 0  i 2 
w.E
33.
asy
The band diagram of a p-type semiconductor with a band-gap of 1 eV is shown.
Using this semiconductor, a MOS capacitor having VTH of − 0.16V, Cox
'
of 100 nF cm2 and a metal

En
work function of 3.87 eV is fabricated. There is no charge within the oxide. If the voltage across the
capacitor is VTH , the magnitude of depletion charge per unit area ( in C cm2 ) is

gin
Vacuum level

eer
4 eV
i ng.
0.5 eV
EC
n et
Ei
E Fs
0.2 eV
EV

(A) 0.93 10−8 (B) 1.4110−8


(C) 0.52 10−8 (D) 1.70 10−8
Key: (D)

© All rights reserved by Thinkcell Learning Solutions Pvt. Ltd. No part of this booklet may be reproduced or utilized in any form without the written permission.

Downloaded From : www.EasyEngineering.net


Downloaded From : www.EasyEngineering.net

|EC-2020|

Sol: In MOS semiconductor

Vaccumlevel

qX i
qX qS
Metal
qm
qB
EC
Eg 2
Ei qP
EF d EF

ww
oxide
EV
Semi − conductor

 w.E
qms = qm − qX +
Eg 
+ qs  ; qs = 1 − 0.7 = 0.3
 2 
= 3.87eV −  4 + 0.5 + 0.3 eV
asy
= 3.87 − 4.8 = −0.93eV

( t
We know that, VTN = Q'SD ( max ) − Q'ss ox + qms + 2qp E) n
ox
Here Q'ss = 0 (there is no charge in oxide )
gin
−0.16 = −0.093 + 0.6 +
QSD eer
QSD
= 0.17
Cox
i ng.
Cox
QSD = 0.17  100  10−9 = 1.7  10−8 n et

34. The base of an npn BJT T1 has a linear doping profile N B ( x ) has shown below.

The base of another npn BJT T2 has a uniform doping NB of 1017 cm−3 . All other parameters are
identical for both the devices. Assuming that the hole density profile is the same as that of doping, the
common-emitter current gain of T2 is

© All rights reserved by Thinkcell Learning Solutions Pvt. Ltd. No part of this booklet may be reproduced or utilized in any form without the written permission.

Downloaded From : www.EasyEngineering.net


Downloaded From : www.EasyEngineering.net

|EC-2020|

1017 ( cm−3 )
NB ( x )

1014 ( cm−3 )

Emitter Base Collector

0 W

ww
(A) approximately 2.0 times that of T1 (B) approximately 2.5 times that of T1

Key: (A) w.E


(C) approximately 0.3 times that of T1 (D) approximately 0.7 times that of T1

35.
asy
A finite duration discrete-time signal x[n] is obtained by sampling the continuous-time signal
x ( t ) = cos ( 200t ) at sampling instants t = n 400, n=0.1,….,7. The 8-point discrete Fourier transform
(DFT) of x[n] is defined as En
7
X k =  x ne
−j
kn
4
, k = 0,1,...,7. gin
eer
n =0

Which one of the following statements is TRUE?


(A) Only X[3] and X[5] are non-zero
(C) Only X[2] and X[6] are non-zero i
(B) Only X[4] is non-zero.
(D) All X[k] are non-zero.
ng.
Key:
Sol:
(C)
It is given that a finite duration discrete signal x(n) is obtained by sampling x ( t ) = cos 200t , with
1
n et
Ts = we need to comment about its 8 point DFT.
400
The discrete time signal is
200 
x ( n ) = cos n = cos n; n = 0,1, 2...7
400 2
  2 3 4 5 6 7 
x ( n ) = cos 0,cos ,cos ,cos , cos ,cos ,cos ,cos 
 2 2 2 2 2 2 2
= 1,0, −1,0,1,0, −1,0

Let y ( n ) = 1, −1,1, −1

The its 4-point DFT will be

© All rights reserved by Thinkcell Learning Solutions Pvt. Ltd. No part of this booklet may be reproduced or utilized in any form without the written permission.

Downloaded From : www.EasyEngineering.net


Downloaded From : www.EasyEngineering.net

|EC-2020|

1 1 1 1   1 
1 − j −1 j   −1
y(k) =     =  0 0 4 0
1 −1 1 −1  1 
  
1 j −1 − j  −1
By comparing x(n) and y(n) we can say
n
x (n) = y 
2
By property
y(n)  Y(k)

wwn
y     Y ( k ) , Y ( k ) , Y ( k ) ...m times 
m
n
2
w.E
y     Y ( k ) , Y ( k ) 

Sin ce y ( n )  0,0, 4,0

n
 
x ( n ) = y    0,0, 4 ,0,0,0, 4 ,0 
2    
asy
 X( 2) X( 6)

So only X (2) and X (6) are non-zero



En
Hence the correct answer is (C).
gin
36. eer
For the modulated signal x(t) = m(t) cos ( 2f c t ) , the message signal m ( t ) = 4cos (1000t ) and the

i
carrier frequency f c is 1MHz, The signal x(t) is passed through a demodulator, as shown in the figure
below. The output y(t) of the demodulator is ng.

Ideal LPF with cut − off 510 Hz
1
y(t)
n et
f ( Hz )
−510 510

cos ( 2 ( fc + 40) t )

(A) cos ( 920 t ) (B) cos ( 460 t ) (C) cos (1000t ) (D) cos ( 540 t )

Key: (A)

© All rights reserved by Thinkcell Learning Solutions Pvt. Ltd. No part of this booklet may be reproduced or utilized in any form without the written permission.

Downloaded From : www.EasyEngineering.net


Downloaded From : www.EasyEngineering.net

|EC-2020|

Sol: s ( t ) = x ( t ) cos ( 2 ( fc + 40) t )

= m ( t ) cos 2f c t.cos ( 2 ( f c + 40 ) t )


1
= m ( t ) cos ( 4f c t ) + 2  40t + cos ( 2  40t )  
2
1
= m ( t ) cos ( 2  40t )
2
= cos (1000t ) cos ( 2  40t )

= cos ( 2 ( 540 ) t ) + ( cos ( 2  460 ) t ) 


1
2
 LPF

ww = cos ( 2  460 ) t = cos ( 920t )

37.
w.E
Consider the following system of linear equations
x1 + 2x 2 = b1; 2x1 + 4x 2 = b2 ; 3x1 + 7x 2 = b3 ; 3x1 + 9x 2 = b4

(A) b3 = 2b1 and 3b1 − 6b3 + b4 = 0


asy
Which one of the following conditions ensures that a solution exists for the above system?
(B) b3 = 2b1 and 6b1 − 3b3 + b4 = 0

(C) b2 = 2b1 and 3b1 − 6b3 + b4 = 0 En (D) b2 = 2b1 and 6b1 − 3b3 + b4 = 0
Key: (D)
gin
Sol: Given, the system of linear equations:
x1 + 2x 2 = b1 eer
2x1 + 4x 2 = b 2
3x1 + 7x 2 = b3 i ng.
3x1 + 9x 2 = b 4

Consider, the augmented matrix


1 b1 
2
n et
2 b 2 
4
 A | B =  3 b3 
7
 
3 b4 
9
R 2 → R 2 − 2R 1 ; R 3 → R 3 − 3R 1 ; R 4 → R 4 − 3R 1 ;

12 b1 
0
0 b 2 − 2b1 
 A | B ~ 0
1 b3 − 3b1 
 
03 b 4 − 3b1 
R 4 → R 4 − 3R 3 ;

© All rights reserved by Thinkcell Learning Solutions Pvt. Ltd. No part of this booklet may be reproduced or utilized in any form without the written permission.

Downloaded From : www.EasyEngineering.net


Downloaded From : www.EasyEngineering.net

|EC-2020|

1 2 b1 
0 0 b 2 − 2b1 
 A | B ~ 0 1 b3 − 3b1 
 
0 0 6b1 − 3b3 + b 4 

1 2 b1 
0 1 b3 − 3b1 
 A | B ~ 0 0 b 2 − 2b1 
→ Echelon form
 
0 0 6b1 − 3b3 + b 4 

 clearly,
(A) = 2

ww
Solution exists for the above system if

b2 = 2b1 w.E
if b2 − 2b1 = 0 and 6b1 − 3b3 + b 4 = 0

 If b 2 = 2b1 and 6b1 − 3b3 + b 4 = 0 then

asy
 ( A ) =  ( AB ) , i.e., solution exists for the above system.

En
38. The magnetic field of a uniform plane wave in vacuum is given by

( )
H ( x, y, z, t ) = a x + 2a y + ba z cos ( t + 3x − y − z ). gin
The value of b is ________. eer
Key:
Sol:
(1)
Given i ng.
H = ( aˆ x + 2aˆ y + baˆ z ) cos ( t + 3x − y − z )( A m )

We have to determine value of “b”.


n et
By comparing the given field with the following general expression

H ( x,y,z ) = ( H x aˆ x + H y aˆ y + H z aˆ z ) cos t − (  x x +  y y +  z z ) 



H

We get H x = 1, H y = 2 and H z = b and x = −3,  y = 1, z = 1

For a UPW H − field H and wave vector “  ” must be mutually perpendicular to each other

i.e., H. = 0 ( i.e., ) ( aˆ x + 2aˆ y + baˆ z ). ( −3aˆ x + aˆ y + aˆ z ) = 0

−3 + 2 + b = 0
b = 1

© All rights reserved by Thinkcell Learning Solutions Pvt. Ltd. No part of this booklet may be reproduced or utilized in any form without the written permission.

Downloaded From : www.EasyEngineering.net


Downloaded From : www.EasyEngineering.net

|EC-2020|

39. For a 2-port network consisting of an ideal lossless transformer, the parameter S21 (rounded off to two
decimal places) for a reference impedance of 10 , is _________.
2 Port Network

2 :1

Port 1 Port 2

Key: ww
(0.8)
Sol: Given,
w.E
2-port network consisting of an ideal lossless transformer


V1 n I
= and 1 =
V2 1 I2 n
1
asy
+
I1

En
2 :1 I2

+
V1
− gin V2

eer
Reference input and output impedances R 01 = R 02 = 10
We have to determine forward transmission coefficient S21 = ?
i ng.
If R 01 = R 02 , then the scattering matrix for an ideal transformer is given by

 2S11
 n −1
S12
2n 

n et
 n 2 + 1 n 2 + 1
S= 
 2n 1 − n2 
 n 2 + 1 n 2 + 1
 S21 S22 
2n 2 2
 S21 = 2 = 2  n = 2
n +1 2 +1
= 0.8
Forward transmission coefficient S21 = 0.8

© All rights reserved by Thinkcell Learning Solutions Pvt. Ltd. No part of this booklet may be reproduced or utilized in any form without the written permission.

Downloaded From : www.EasyEngineering.net


Downloaded From : www.EasyEngineering.net

|EC-2020|

40. P, Q and R are the decimal integers corresponding to the 4-bit binary number 1100 considered in signed
magnitude, 1’s complement, and 2’s complement representations, respectively. The 6-bit 2’s
complement representation of (P+Q+R) is
(A) 110101 (B) 111001 (C) 110010 (D) 111101
Key: (A)
Sol: It is given that
P represent decimal equivalent of signed magnitude number: 1100
Q represent decimal equivalent of 1’s complement number: 1100
R represent decimal equivalent of 2’s complement number: 1100
We need to obtain 6 digits, 2’s complement form of P + Q + R.

ww
1100 in signed magnitude form = –[Decimal equivalent of (binary 100)] = –4
So, P = –4.

w.E
1100 in 1’s complement form
= – [Decimal equivalent of (1’s complement of 1100)]
= – [Decimal equivalent of 0011]
= –3 asy
So, Q = –3
En
1100 in 2’s complement form
= – [Decimal equivalent of (2’s complement of 1100)] gin
= – [Decimal equivalent of 0100]
= –4 eer
So, R = –4
P + Q + R = (–4) + (–3) + (–4) = (–11)10
i ng.
+ 11 → 01011 (in 2’s complement form)
– 11 → 10101 (in 2’s complement form)
n et
Now to increase the bit size to 6 from 5 bits.
We need to copy the sign bit once towards left.
So, 6 bit 2’s complement representation is 110101.
Hence the correct answer is 110101.

© All rights reserved by Thinkcell Learning Solutions Pvt. Ltd. No part of this booklet may be reproduced or utilized in any form without the written permission.

Downloaded From : www.EasyEngineering.net


Downloaded From : www.EasyEngineering.net

|EC-2020|

41. The components in the circuit given below are ideal. If R = 2k and C = 1 F, the -3 dB cut-off
frequency of the circuit in Hz is
R

Vi ( j) R
C

Vo (j)
+
2C 2R

ww
Key:
(A) 79.58
(A) w.E (B) 34.46 (C) 14.92 (D) 59.68

asy
Sol: Given the following circuit

En
gin
R I1 I2
Vi − C
N 0A
Vo

I = 0A 2R 2C P
0A
+
eer
i ng.
From virtual short concept VN = VP = 0V
n et
The current flowing through parallel combination of 2R and 2C is
VN
I= = 0A  VN = 0V
 1 
 2R || 
 2sC 
Applying KCL at node “N” we get

© All rights reserved by Thinkcell Learning Solutions Pvt. Ltd. No part of this booklet may be reproduced or utilized in any form without the written permission.

Downloaded From : www.EasyEngineering.net


Downloaded From : www.EasyEngineering.net

|EC-2020|

I1 = I 2
Vi − VN VN − Vo  1 R 
=  Where Z = R || sC = sCR + 1 
R Z  
V −Z
 o =  VN = 0V
Vi R
R

V
 o = sCR + 1 = −1
Vi R sCR + 1
Put s = j

Vo 1 1
 = =

ww Vi 1 + jRC 1 + ( RC )
2

Vo 1
At 3-dB cut off frequency ( f c ) ,
w.E1 1
Vi
=
2

(i.e.,)
1 + ( C RC )
2
=
2
asy
C =
1
RC
rad sec
En
Cut off frequency f c =
1
=
1
2RC 2  2  103  10−6
= 79.58Hz gin
Hence the correct option is (A)
eer
42. i
Using the incremental low frequency small-signal model of the MOS device, the Norton equivalent
ng.
n
resistance of the following circuit is
VDD

R
et
g m, rds

rds + R 1
(A) (B) rds + R + gm rds R (C) rds + +R (D) rds + R
1 + g m rds gm

© All rights reserved by Thinkcell Learning Solutions Pvt. Ltd. No part of this booklet may be reproduced or utilized in any form without the written permission.

Downloaded From : www.EasyEngineering.net


Downloaded From : www.EasyEngineering.net

|EC-2020|

Key: (A)
Sol: The small signal equivalent model for the given circuit can be represented as follows

G D id
+

gs g m gs rds  i1 R

− S id

R in
Ix loop (1)

ww Vx +

Vx = −gs
w.E
 R in =
Vx asy
…(1)

…(2)
Ix
Applying KCL at node “S” we get I x + id = 0 En
Apply KVL around loop (1) we get gin
…(3)

0 − id R − i1rds − Vx = 0

Where i1 = ( id − gmgs ) = ( id + gm Vx )
eer
− Vx = id R + ( id + g m Vx ) rds i ng.
id =
−Vx (1 + g m rds )
R + rds
Substituting “id” in equation (3) we get
n et
Vx R + rds
= R in =
Ix 1 + g m rds
R + rds
Hence the equivalent resistance looking into source terminal is R in =
1 + g m rds
Hence the correct option is (A).

© All rights reserved by Thinkcell Learning Solutions Pvt. Ltd. No part of this booklet may be reproduced or utilized in any form without the written permission.

Downloaded From : www.EasyEngineering.net


Downloaded From : www.EasyEngineering.net

|EC-2020|

43. For the BJT in the amplifier shown below, VBE = 0.7V, kT q = 26 mV. Assume that BJT output
resistance ( ro ) is very high and the base current is negligible. The capacitors are also assumed to be
short circuited at signal frequencies. The input Vi is direct coupled. The low frequency voltage gain
v0 vi of the amplifier is
VCC = 10V

RC
10 k
V0
C1

ww 1 F RL
10 k

w.E Vf

asy RE
20 k
CE
100 F

VEE = −10V En
gin
Key:
(A) –178.85
(D)
(B) –128.21
eer
(C) -256.42 (D) -89.42

Sol: Small signal equivalent model for the given circuit is shown in following figure
i ng.
Vi ~ be
+
r
ic

( R C || R L )
+ n et
gm be Vo

be = Vi
V0 = −i c ( R C || R L ) = −g m be ( R C || R L )

Vo
 = −g m ( R C || R L ) → (1)
Vi

© All rights reserved by Thinkcell Learning Solutions Pvt. Ltd. No part of this booklet may be reproduced or utilized in any form without the written permission.

Downloaded From : www.EasyEngineering.net


Downloaded From : www.EasyEngineering.net

|EC-2020|

DC equivalent model for the given circuit is


VCC = 10V
10k

C
B
+
0.7 −
E
Loop (1) 20k IE

ww VEE = −10V

w.E
Applying KVL around loop (1) we get
0 − 0.7 − I E ( 20k ) + 10 = 0
 I E = 0.465mA

 =  implies IC = IE = 0.465 mA asy


gm =
IC
Where VT = 26mV En
gin
VT
= 38.46  0.465 10−3 = 17.8m
Substituting value of “ g m ” in equation (1) we get

Vo
= −17.88  10−3 10k ||10k 
 R C = 10k and  eer
Vi
= −89.42

R L = 10k


i ng.
Hence the correct option is (D).
n et
44. In a digital communication system, a symbol S randomly chosen from the set s1 ,s 2 ,s3 ,s 4  is
transmitted. It is given that s1 = −3, s2 = −1, s3 = +1 and s 4 = +2. The received symbol is T = S + W.

W is a zero-mean unit-variance Gaussian random variable and is independent of S. Pi is the conditional


probability of symbol error for the maximum likelihood (ML) decoding when the transmitted symbol
S = si . The index if for which the conditional symbol error probability Pi is the highest is ______.
Key: (3)
Sol: ML Decision rule for AWGN channel
No
The ML decision rule can be simplified as, here i = m,
ˆ = 1,  = 0
2

© All rights reserved by Thinkcell Learning Solutions Pvt. Ltd. No part of this booklet may be reproduced or utilized in any form without the written permission.

Downloaded From : www.EasyEngineering.net


Downloaded From : www.EasyEngineering.net

|EC-2020|

m̂ = arg max  P ( r s m̂ ) 

= arg max l n ( P ( r s m̂ ) )

 −N 1 N
2
= arg max  l n ( N o ) − r k − s mk
ˆ 

 2 No k =1 
 4 2
= arg min  rk − s mk ˆ 

 k =1 


= arg min r − s m̂

2

We select that vector sm which has minimum Euclidean distance

ww s1 2 s2 2 s3 1 s4

−3
w.E−1 1 2

We can choose

asy
s 3 , such that it has minimum Euclidean distance m = 3 = i

En
gin
dy
45. Which one of the following options contains two solutions of the differential equation = ( y − 1) x ?
dx
(A) ln y − 1 = 2x 2 + C and y = 1

(C) ln y − 1 = 0.5x 2 + C and y = 1 eer


(B) ln y − 1 = 2x 2 + C and y = −1

(D) ln y − 1 = 0.5x 2 + C and y = −1

Key: (C)
i ng.
n
dy
Sol: Given D.E is = ( y − 1) x


1
y −1
dx

dy = xdx et
Variable − separable D.E
1
 dy =  xdx
y −1
x2
 n y −1 = +c
2
D.E is n y − 1 = 0.5x 2 + c
Also y = 1 will satisfy given D.E.

© All rights reserved by Thinkcell Learning Solutions Pvt. Ltd. No part of this booklet may be reproduced or utilized in any form without the written permission.

Downloaded From : www.EasyEngineering.net


Downloaded From : www.EasyEngineering.net

|EC-2020|

46. The current I in the given network is

Z
120 − 90 V +

I
Z = ( 80 − j35 ) 


120 − 30 V + Z

ww
(A) 2.38 − 23.63 A. (B) 2.38143.63A

Key: (B) w.E


(C) 2.38 − 96.37A (D) 0A.

Sol: The given circuit is as shown below


asy
120 − 90 ~
+
En +
Z
− −

gin A
I1
Z = 80 − j35

120 − 30 ~
− Io −
eer
i
Z
+
+ I2
ng.
We need to obtain Io
By KCL at node A
120 − 90 120 − 30 
n et
Io = − ( I1 + I2 ) = −  + = 2.38 −36.3
 80 − j35 80 − j35 

So, magnitude of Io = 2.38


Hence the correct answer is 2.38143.63A

1
47. A system with transfer function G ( s ) = , a  0 is subjected on an input 5cos 3t. The steady
( s + 1)( s + a )
1
state output of the system is cos ( 3t − 1.892) . The value of a is ________.
10
Key: (4)

© All rights reserved by Thinkcell Learning Solutions Pvt. Ltd. No part of this booklet may be reproduced or utilized in any form without the written permission.

Downloaded From : www.EasyEngineering.net


Downloaded From : www.EasyEngineering.net

|EC-2020|

Sol: Given,
1
Input = 5cos(3t) and output = cos ( 3t − 1.892 )
10
1
We need to find a for G ( s ) =
( s + 1)( s + a )
1 5 1
 =
10 10 9 + a 2
 9 + a 2 = 25  a = 4
Hence, the correct value of ‘a’ is 4.

48. ww
For the solid S shown below, the value of  xdxdydz (rounded off to two decimal places) is_____.

w.E
S
z

asy 1

En0
1
y

gin
3 eer
x i ng.
Key:

Sol:
(2.25)

   x dxdy dz =  ( xdx ) A
0
3

 x2 
3
z

1
n et
= 0.5   1 A =  1  1 = 0.5
2
 2 0
9
= 0.5  0 y
2 1
= 2.25

Hence the value of  x dxdy dz is 2.25


3
x

© All rights reserved by Thinkcell Learning Solutions Pvt. Ltd. No part of this booklet may be reproduced or utilized in any form without the written permission.

Downloaded From : www.EasyEngineering.net


Downloaded From : www.EasyEngineering.net

|EC-2020|

49. A pn junction solar cell of area 1.0 cm2 , illuminated uniformly with 100 mW cm−2 , has the following
parameters: Efficiency = 15%, open circuit voltage =0.7V, fill factor = 0.8, and thickness = 200m. The
charge of an electron is 1.6 10−9 C. The average optical generation rate ( in cm−3s−1 ) is

(A) 1.04 1019 (B) 0.84 1019 (C) 5.57 1019 (D) 83.60 1019
Key: (B)
Sol: We know that in solar cell,
Vm Im
Fill factor = = FF
Isc Voc
Vm Im

ww
=
Pin
 100

FF  Voc .Isc
So,  =

15 =
w.E
Pin
0.8  0.7Isc
 100
 100

100m  1
15
Isc =  10−1  A asy
56
Optical generation rate can be calculated by En
J sc =
Isc
A
t
= e  G L dt, where t = thickners gin
eer
0

= eG L .t

Isc = AeG L t

GL =
Isc i ng.
e t
15
= 
10 −1
56 1.6  10 −19  200  10 −4
n et
15 10 −1
= 
56 16  2  10 −5  10 2  10 −19
15  100
=  1019
56  32
= 0.837  1019 cm −3s −1
= 0.84  1019 cm −3s −1

© All rights reserved by Thinkcell Learning Solutions Pvt. Ltd. No part of this booklet may be reproduced or utilized in any form without the written permission.

Downloaded From : www.EasyEngineering.net


Downloaded From : www.EasyEngineering.net

|EC-2020|

50. SPM ( t ) and SFM ( t ) as defined below, are the phase modulated and the frequency modulated waveforms,
respectively, corresponding to the message signal m(t) shown in the figure.
SPM ( t ) = cos (1000t + K p m ( t ) )

(
and SFM ( t ) = cos 1000t + K f  m (  ) d
t

− )
Where K p is the phase deviation constant in radians/volt and K f is the frequency deviation constant in
radians/second/volt. If the highest instantaneous frequencies of SPM ( t ) and SFM ( t ) are same, then the
Kp
value of the ratio is ____________ seconds.
Kf

ww V
10
m(t)

w.E
asy
0
1 3
En 6 7 t ( sec )

Key: (2)
gin
Sol: The PM and FM equations are shown below
SPM = A c cos 1000t + k p m ( t )  eer


t

SFM = A c cos 1000t + k f  m ( t ) dt 

i ng.
n
−

Given,
m(t) et
10

1 3 6 7 → t ( sec )

kp d
In PM f i ( max ) = 500 + ( m(t) )
2 dt max

© All rights reserved by Thinkcell Learning Solutions Pvt. Ltd. No part of this booklet may be reproduced or utilized in any form without the written permission.

Downloaded From : www.EasyEngineering.net


Downloaded From : www.EasyEngineering.net

|EC-2020|

kf
In FM fi( max ) = 500 + m(t)
2 max

Both are equal when


kp d k
m(t) = f m(t)
2 dt 2 max

kp 10
= =2
kf 5

51. For an infinitesimally small dipole in free space, the electric field E  in the far field is proportional to
(e / r ) sin , where k = 2 . A vertical infinitesimally small electric dipole ( l   ) is placed at a
ww − jkr

distance h (h>0) above an infinite ideal conducting plane, as shown in the figure. The minimum value of

w.E
h, for which one of the maxima in the far field radiation pattern occurs at  = 60, is
Z

asy Il
En
gin
h

eer
0 i y

ng.
Key:
Sol:
(A) 0.5
(B)
Given,
(B)  (C) 0.25 (D) 0.75
n et
An infinite & small dipole ( l   )

e − jkr 2
E sin  where k =
r 
We have to determine minimum value of “h” for which one of the maxima in the far field radiation
pattern occurs at  = 60

© All rights reserved by Thinkcell Learning Solutions Pvt. Ltd. No part of this booklet may be reproduced or utilized in any form without the written permission.

Downloaded From : www.EasyEngineering.net


Downloaded From : www.EasyEngineering.net

|EC-2020|

I 0o

h
 = 60
d = 2h
y

Infinite conducting plane


h

ww Image

w.E I 0o

asy
Current phase shifts between Idl and its image is  = 0o
When ever we place a dipole at height “h” above the conducting plane, there forms its image at a depth

En
“h” below the conducting plane and thus it forms a two-element array.
For a 2-element array, array factor is given by

A.F = 2cos  
2 gin
Array factor is maximum when

=  m i.e.  = 2m eer
Where  = kdcos  +  = 2m
2
i ng.
Where  = 0 and k =

2
2

and we want maximum at  = 60o

d cos 60o + 0 = 2mm = 0,1, 2....


n et

d
= m
2
d = 0, 2, 4,..........
Minimum value of d = 2
 d = 2
h = 
Minimum value of h = 

© All rights reserved by Thinkcell Learning Solutions Pvt. Ltd. No part of this booklet may be reproduced or utilized in any form without the written permission.

Downloaded From : www.EasyEngineering.net


Downloaded From : www.EasyEngineering.net

|EC-2020|

K ( z − )
52. The transfer function of a stable discrete-time LTI system is H ( z ) = where K and  are real
z + 0.5
numbers. The value of  (rounded off to one decimal place) with   1, for which the magnitude
response of the system is constant over all frequencies, is _________.
Key: (-2)
k ( z − )
Sol: The transfer function of a stable discrete time system is H ( z ) = , where   1, if magnitude
z + 0.5
response is constant for all frequency we need to obtain .
Magnitude response is constant for all frequency means H(z) represent transfer function of an all pass
filter.

ww  1 *
In an all pass filter Po is pole then its zero must be   .
 Po 
Here Po = −0.5
w.E*
1  1 

asy
*

So  =   =   = ( −2 ) = −2
*

 Po   −0.5 
So, value of  = −2.
Hence the correct answer is –2. En
gin
53.
eer
X is a random variable with uniform probability density function in the interval  −2,10.For Y = 2X − 6,
the conditional probability P ( Y  7 X  5) (rounded off to three decimal places) is ________.

Key: (0.3)
i ng.
n
Sol: Pdf of X

1
12 et
−2 10 → X

Given, Y = 2X − 6
Y in range  −10 14 ,

1
24

−10 14 Y
© All rights reserved by Thinkcell Learning Solutions Pvt. Ltd. No part of this booklet may be reproduced or utilized in any form without the written permission.

Downloaded From : www.EasyEngineering.net


Downloaded From : www.EasyEngineering.net

|EC-2020|

P ( Y  7  X  5)
P ( Y  7 X  5) =
P ( X  5)

Y+6
X= 5
2
y + 6  10
y4

=P
(Y  4  Y  7)
P ( X  5)
7
1
 24 dY 3

ww
= 104
1
5 12 dX
= 24
5
12

=
3
10
= 0.3
w.E
54. asy
For the components in the sequential circuit shown below, t pd is the propagation delay, t setup is the setup

En
time, and t hold is the hold time. The maximum clock frequency (rounded off to the nearest integer), at
which the given circuit can operate reliably, is _______MHz.

gin
Flip Flop1 eer
t pd = 3 ns i t pd = 2ns

ng.
t pd = 2ns

Clk
t setup = 5 ns
t hol = 1ns
IN
n et
t pd = 8 ns
t setup = 4 ns
t hol = 3ns

Key: (76.92)

© All rights reserved by Thinkcell Learning Solutions Pvt. Ltd. No part of this booklet may be reproduced or utilized in any form without the written permission.

Downloaded From : www.EasyEngineering.net


Downloaded From : www.EasyEngineering.net

|EC-2020|

Sol: The given circuit diagram is as shown below

I1
t pd = 2ns
Y1 t pd = 2ns
t pd = 3 ns
( I2 )
t setup = 5 ns
t hold = 1ns
CLK
 FF1 IN

ww t pd = 8 ns
Y2
I2
w.E t setup = 4 ns
t hold = 3ns

asy
 FF2

En
We need to obtain the CLK frequency for reliable operation.
gin
Set up time: ( t s ) : Flip flop inputs ( I1 , I 2 ) should be stable at least for t s interval, before arrival of
clock.
eer
Let as assume all states are stable and a clock trigger arrives.
i
Hold time: ( t h ) : Flip flop inputs ( I1 , I 2 ) should be stable at least for t h interval after arrival of clock.

ng.
(i) Y1 will be available at 3nsec (Q t pd1 = 3n sec )

(ii) Y2 will be available at 8nsec Q t pd2 = 8nsec ( )


n et
(iii) I 2 will be available after delay of XOR, NAND gates since Y, is available at 3nsec we will get I 2
after 3 + 2 + 2 = 7nsec (delay of both logic gates).
(iv) I1 will be available when Y2 will be available i.e., at 8nsec.
Finally, if all states are stable and a clock arrives, then to process the flip flops i.e. the new inputs to flip
flop are available at I1 = 8nsec, I2 = 7nsec
But before arrival of next clocks these new I1 , I2 should be stable before there corresponding flip flops
set up time.

So, for flip flop 1, the clock period should be

© All rights reserved by Thinkcell Learning Solutions Pvt. Ltd. No part of this booklet may be reproduced or utilized in any form without the written permission.

Downloaded From : www.EasyEngineering.net


Downloaded From : www.EasyEngineering.net

|EC-2020|

8nsec+ t s1 = 8nsec+ 5nsec = 13nsec


For flip flop 2 the clock period should be
7n sec+ t s2 = 7n sec+ 4n sec = 11n sec

Since both flip flops are simultaneously triggered from a single clock, for reliability on both flip flops.
TCLK  max (13n sec,11n sec ) = 13n sec

(If we take 11nsec, FF1 will have errors)


1 1
f CLK   f CLK   f CLK  76.92 MHz
TCLK 13 10−9

ww
So maximum value of clock frequency can be 76.92 MHz
Hence the correct answer is option 76.92.

55.
w.E
An enhancement MOSFET of threshold voltage 3 V is being used in the sample and hold circuit given

asy
below. Assume that the substrate of the MOS device is connected to -10V. If the input voltage VI lies
between 10V, the minimum and the maximum values of VG required for proper sampling and holding
respectively, are
En
V1
gin V0

eer
VG
i ng.
(A) 3V and -3V (B) 13V and -7V (C) 10V and -10V
n
(D) 10 V and -13 V et
Key: (B)
Sol: Given following circuit
V1 V0

VG

VTh = 3V, Vsub = −10V; V1 : ( −10V + 10V )

© All rights reserved by Thinkcell Learning Solutions Pvt. Ltd. No part of this booklet may be reproduced or utilized in any form without the written permission.

Downloaded From : www.EasyEngineering.net


Downloaded From : www.EasyEngineering.net

|EC-2020|

 During sampling VGS  VTH (MOSFET must be ON)

 Vs = V1max = 10V 
( i.e ) VG − Vs  VTh  max 
and VTh = 3V 
max

 VG − 10  3
 VG  13V ...(1)

 During hold VGs  VTh (MOSFET is in OFF state)

 Vsmin = V1min = −10V 


( i.e ) VG − VS  V  
& VTh = 3V
Th

min

 VG − ( −10 )  3

ww
 VG  −7V ...( 2 )

 From (1) & (2) we can say option (B) is correct

w.E
asy
En
gin
eer
i ng.
n et

© All rights reserved by Thinkcell Learning Solutions Pvt. Ltd. No part of this booklet may be reproduced or utilized in any form without the written permission.

Downloaded From : www.EasyEngineering.net

You might also like